Are you seeking one-on-one college counseling and/or essay support? Limited spots are now available. Click here to learn more.

How to Write the AP Lang Synthesis Essay with Example

September 5, 2023

If you’re highly interested in learning more about writing analysis, then chances are you enrolled in AP Lang. Essentially, AP Lang is an advanced course for high schoolers that combines interest and knowledge in English with critical thinking. In the class, students learn how to analyze and synthesize a variety of texts to construct well-reasoned arguments. If you take AP Lang, then you can opt to take the AP test at the conclusion of the school year. On the exam, students write the AP Lang synthesis essay to demonstrate their learned abilities. In this article, we’ll look at what the AP Lang synthesis essay requires and show an example to provide better understanding of what to expect on the exam.

AP Lang Exam Basics

The AP Lang exam is separated into two sections. In the first section, students have one hour to answer a series of 45 multiple-choice questions. Here, about half of the questions are based on passages students read. The other half are focused on the best revision techniques. Essentially, the answers for the latter 20-22 questions are geared toward revising mock essays.

In this article, however, we’ll focus mainly on the second part of the exam: the AP Lang synthesis essay.

In this second section, students have two hours and 15 minutes to write three essays of their own design. The three open-ended questions in this section are intended to be free-response and allow for a variety of approaches. Each question is intended to allow up to 40 minutes to complete.

For the AP Lang synthesis essay, students are presented with a scenario of the College Board’s design. The scenario will provide its own thesis statement. Usually, scenarios relate to real-world problems like environmental concerns, media, or government policies.

For each scenario, students are provided with 6-7 outside sources. These sources could be in the form of an image, visual graph, or written paragraph. For written paragraphs, the sources are usually no more than 500 words.

Students are then expected to incorporate at least 3-4 of these outside sources into their essay response. The outside sources are intended to be used as supporting evidence for the student’s chosen stance or argument. Students are able to either agree with or disagree with the thesis presented in the original scenario.

AP Lang Exam – Scoring

In the second part of the AP Lang exam, students can earn a possible 6 points on each essay. 1 point is earned for the development of a thesis. Up to 4 points can be earned for evidence and commentary. The final 1 point is earned for sophistication of thought.

AP Lang Exam – Takeaways

Ultimately, the goal of the AP Lang synthesis essay is not whether the student is “right” or “wrong” in their argument. The key is that students are able to reasonably and clearly support their argument using the provided sources as evidence .

The College Board looks for your ability to identify relationships between texts , form a coherent argument , and interpret external sources .

Synthesis Essay AP Lang Examples

If you’re not sure how the questions will look on the AP Lang synthesis essay section, we’ll provide an example. After the example, we’ll break down the strengths and weaknesses of the response. That way, you’ll have a better idea of what the College Board is looking for.

Additionally, the College Board has released previous AP Lang synthesis essay examples you can review. They even have essay questions as recent as 2022 . For further support, a scoring commentary and comments from the Chief Reader are also available to view. Additionally, there are other examples you can view from earlier years .

Note: A good strategy to study for the synthesis essay AP Lang exam is to review your rhetorical devices and literary devices . Understanding how these devices function can be essential in constructing a cohesive essay.

Synthesis Essay AP Lang Examples – Sample Question

Below is a sample question from the AP Lang synthesis essay and a response to the prompt. This question was taken directly from a 2022 exam . However, the response to the question will be originally crafted for the purpose of this newsletter. As well, all supporting evidence will be originally created and does not correspond to any previous test.

The Question

Since the early 2000s, the United States government and a number of corporations have sponsored initiatives to improve education in the STEM disciplines: science, technology, engineering, and mathematics. The emphasis on STEM subjects in elementary, secondary, and higher education reflects concerns that United States students are less proficient in these areas than are students in other countries. Additionally, there is a belief that mastery in STEM fields is now essential in order to join a highly technical and specialized workforce. However, not everyone is convinced that a STEM-focused curriculum is necessary and/or effective.

In your response you should do the following:

  • Respond to the prompt with a thesis that presents a defensible
  • Select and use evidence from at least three of the provided sources to support your line of Indicate clearly the sources used through direct quotation, paraphrase, or summary. Sources may be cited as Source A, Source B, etc., or by using the description in parentheses.
  • Explain how the evidence supports your line of
  • Use appropriate grammar and punctuation in communicating your

How to Approach the Question

Maybe your first thought upon seeing this block of text is to feel overwhelmed. But don’t panic. There are effective ways to approach the question so you will be more prepared in your response.

It’s a good strategy to first isolate the thesis . What is the main idea of the text, and what is its argument?

Try it out. Reread the prompt and see if you can identify what the statement is asking you to develop an opinion on.

Think you’ve got it? In this example, we will be focusing on whether or not a STEM-focused curriculum in K-12 education is necessary and/or effective. In short, we will be arguing either for (highlighting the benefits) or against (highlighting the pitfalls) a STEM-focused curriculum.

How do we know what this statement is asking us?

Well, the statement provides a lot of background information. For example, we receive a definition of what STEM stands for. As well, we know that since 2000, there has been a greater initiative for STEM-focused classes.

When you read the prompt for the first time, it’s a great strategy to learn how to differentiate between background and contextual information from the heart of the argument .

A good way to learn how to isolate the argument is to look for transition words. Usually, these appear near the end of the question. Words like “however” and “yet” are signals that the statement is offering a differing opinion. Typically, the statement will tell you which two positions it’s offering for argument. These opinions are usually signaled by contrasting transition words.

So, now that we know what the question is asking us, what is the best way to respond?

Synthesis Essay AP Lang Examples – Sample Answer

The following is an essay response I crafted to the above question. After reading the sample, I will break down what it does well and what areas can be improved.

A STEM-focused curriculum is not as essential to providing a meaningful K-12 education. Because the majority of high school students are not proficient in STEM-focused classes, prioritizing these classes causes harm to student’s mental health and academic performance.

As seen in Source A, 60% of high school seniors in the Midwest only scored a C average in math and science-based classes (Langston). This statistic suggests that the majority of students do not resonate with STEM classes and therefore perform poorly. Earning a low score in any class does not bode well for students’ mental health.

When looking at the primary argument in Source C, it’s clear that most high schoolers prefer creative outlets to fact-based research (Kohler). Allowing students the opportunity to be more creative and initiate conversations about coursework lets students be more active in their learning. When students can discuss the nuance in their opinions, more personal growth happens. These conversations are not always easy to have in STEM-focused classes.

As well, when looking back to Source A, it’s clear that high school students in the Midwest earned higher grades, on average, in their English and art classes (Langston). This figure suggests that students perform better in these classes because they relate more to the source material. When relating to what they learn, they perform better in class.

In conclusion, STEM-focused curriculum is not as essential in K-12 education because most high school students do not relate to their STEM classes. When students do not earn satisfactory grades in these classes, it negatively affects their future college applications and job prospects.

Synthesis Essay AP Lang Examples – Answer Breakdown

So, what does this essay response get right, and where can it be improved? Let’s start with what the response does well.

First, the response establishes its thesis right away. Usually, it’s a good idea to clearly state your argument within the first paragraph. Not only is this a good practice because a reader can easily identify your stance, but also you can refer to your thesis as you write to make you stay on track.

With your thesis, it’s also a good idea to include one to two supporting sentences with the reasons why the thesis is concluded . Like in this example, I wrote that STEM-focused classes should not be prioritized because they can negatively affect both mental health and academic performance.

Another positive aspect of this response is that it is sure to not only reference but also cites its sources . It’s important that the reader understand where your information is coming from. That way, the readers can ensure you are interpreting the sources correctly.

AP Lang Synthesis Essay (Continued)

However, when rereading the instructions, it’s clear that this response fails the basic requirement of referring to at least three sources. Always make sure to reread the instructions to ensure you meet the standard requirements for incorporating source material.

Further, this AP Lang synthesis essay does not fully support its arguments . Ideas are simply stated and are not expanded upon.

For example, I mentioned a few times that earning low grades in STEM classes leads to negative mental health for high school students. However, there is no source referenced that either confirms or denies this claim. Therefore, there is no sufficient evidence to support my argument. It relies purely on inference.

Additionally, this AP Lang synthesis essay does not arrive at a sufficient level of sophistication of thought . Basically, sophistication of thought means avoiding broad generalizations and vague claims. The more specific you can be, the better your argument will sound.

Synthesis Essay AP Lang – In Conclusion

In the end, it’s always helpful to read the prompt thoroughly before writing. As well, making notes while you read could be a good strategy to pinpoint main ideas both in the prompt and the sources. That way, you can reread the material quickly. Similarly, sketching an outline may also be helpful. In addition, you should always carefully read the instructions to ensure all guidelines are followed.

As long as you avoid broad generalizations and use enough supporting evidence for your claim, you will be on the right path!

  • High School Success

Meghan Dairaghi

With a BA in English and an MFA in Creative Writing, Meghan has served as a writing tutor at the University of Missouri St. Louis and Maryville University. Additionally, Meghan has held editorial roles at River Styx and Boulevard, and was a prose reader at Farside Review . Most recently, her work has been featured in Belle Ombre , Flypaper Lit , and Mag 20/20 , among others, and she was nominated for the Mary Troy Prize in Fiction. 

  • 2-Year Colleges
  • Application Strategies
  • Best Colleges by Major
  • Best Colleges by State
  • Big Picture
  • Career & Personality Assessment
  • College Essay
  • College Search/Knowledge
  • College Success
  • Costs & Financial Aid
  • Data Visualizations
  • Dental School Admissions
  • Extracurricular Activities
  • Graduate School Admissions
  • High Schools
  • Law School Admissions
  • Medical School Admissions
  • Navigating the Admissions Process
  • Online Learning
  • Private High School Spotlight
  • Summer Program Spotlight
  • Summer Programs
  • Teacher Tools
  • Test Prep Provider Spotlight

“Innovative and invaluable…use this book as your college lifeline.”

— Lynn O'Shaughnessy

Nationally Recognized College Expert

College Planning in Your Inbox

Join our information-packed monthly newsletter.

logo-type-white

AP® English Language

How to ace the ap® english language and composition synthesis essay.

  • The Albert Team
  • Last Updated On: March 1, 2022

how_to_ace_the_ap_english english language synthesis essay

The newest section of the AP® English Language and Composition Exam, the synthesis essay, is one of three essays you will be completing during the examination’s 2-hour free-response period. However, you’ll also have a 15-minute reading and planning period just for this essay, and if you use this time to plan effectively, you can’t go wrong.

Before we get into specific advice on how to handle the AP® English Language and Composition synthesis essay, you need to know what this part of the test really is. It is very similar to the argumentative essay you will also write as part of this exam, except that you are provided with a wealth of source material from which to draw some support for your ideas.

While this in some ways makes the AP® English Language and Composition synthesis essay easier than the argument essay (because you can use quotations, point to authoritative sources for support, etc.), there is an extra element of complexity, and the AP® readers want to see how well you can sort through your source material and put it to good use – which makes planning all that much more important. This brings us to our first tip…

1. Use Your 15-Minute Planning Period Wisely.

The main purpose of this 15-minute period is to give you time to read the source materials. This essay will present you with several sources providing different information about or opinions on a certain topic. Make sure you don’t just skim them, but read them closely – make notes, underline key sections you may want to quote later, etc.

You should also begin outlining your essay and considering your opinion on the subject; have this opinion in mind before you start writing the essay, as you will use it to construct your thesis.

You’ve already learned how to structure persuasive essays in this class and in other classes you have taken; put that knowledge to good use now, and have your main points set out before you start writing. Try to have a thesis statement written by the time you start the essay – your thesis should establish your opinion and the general reasons you feel this way; the rest of your essay will go on to justify and exemplify these reasons. Also write down some of the main points upon which you will base subsequent paragraphs and mark quotes or sections of the sources you can use in each of these paragraphs.

2. Evaluate Your Sources.

ap exams score

Every source you can use for the AP® Language and Composition synthesis essay will have a small box above it explaining where it comes from and who said it – to see exactly what this looks like, check out the free synthesis essay sample questions at AP® Central. There are also public sample questions available there for the rest of the AP® English and Composition Exam .

Keep all information about your sources in mind when you’re quoting them or using them to support your arguments. What journal an article appeared in can say a great deal about its potential biases. For example, consider a question on the environmental impacts of corporate practices – an environmental journal is obviously going to be biased in favor of more environmental regulation, while a report from a company spokesperson will probably gloss over some of the negative impacts of his company. Think critically.

3. Keep Your Tone Consistent.

There is no hard-and-fast advice about what tone you should take – some students try to inject a little humor into their essays while others prefer to be as serious as possible, some are extremely critical and others more accepting. However, the one thing you really have to do while writing the AP® Language and Composition synthesis essay (or any other essay) is keep your tone consistent. Jot some tone-related ideas down as you outline during the 15-minute reading period, and keep in mind everything you’ve learned about tone and other aspects of rhetoric so far this year.

4. Use Rhetorical Technique to Your Advantage!

The various rhetorical practices you’ve been learning about all year can be put to good use here. This class and this test aren’t just about recognizing and analyzing these techniques when others use them, but about preparing you for college and your career by teaching you how to use them effectively yourself. However, this isn’t just about writing a beautiful essay, so read on to Tip # 5!

5. Your Argument Must be Well-Crafted.

The AP® English Language and Composition Exam synthesis essay does not have right or wrong answers; rather, it asks you for your opinion. The AP® Examiner cannot take points off because she disagrees with you. However, you must show logical basis for your opinion, drawing on both the sources AND your own knowledge and experience.

To do this, make sure you have a clear and complete thesis. Make sure the ideas expressed in the beginning of each paragraph or section support the thesis, and that you in turn show how those ideas are supported by a source or through your own knowledge and experience. Don’t generalize or write anything down that you can’t support.

Looking for AP® English Language practice?

Kickstart your AP® English Language prep with Albert. Start your AP® exam prep today .

Interested in a school license?​

Popular posts.

AP® Physics I score calculator

AP® Score Calculators

Simulate how different MCQ and FRQ scores translate into AP® scores

synthesis essay 2022 ap lang

AP® Review Guides

The ultimate review guides for AP® subjects to help you plan and structure your prep.

synthesis essay 2022 ap lang

Core Subject Review Guides

Review the most important topics in Physics and Algebra 1 .

synthesis essay 2022 ap lang

SAT® Score Calculator

See how scores on each section impacts your overall SAT® score

synthesis essay 2022 ap lang

ACT® Score Calculator

See how scores on each section impacts your overall ACT® score

synthesis essay 2022 ap lang

Grammar Review Hub

Comprehensive review of grammar skills

synthesis essay 2022 ap lang

AP® Posters

Download updated posters summarizing the main topics and structure for each AP® exam.

synthesis essay 2022 ap lang

How to Write the AP Lang Synthesis Essay

synthesis essay 2022 ap lang

AP Lang test is the logical conclusion to the introductory college English composition course. And its most important (and often difficult) part is the AP Lang synthesis essay. Despite it being the very basic layer of your future composition skills, it’s a very complicated challenge to approach unprepared. Besides, it's details may change year to year. So let’s have a look with our coursework writing services team at what your AP Lang exam 2022 might look like.

What is AP Lang?

AP Lang is a relatively lengthy test. There are several AP rubrics that a student must be well-versed in to hope to pass it. The first section includes reading and writing, while the second is slightly more freeform and includes three different types of essays.

Among those three, the most interesting and, coincidentally, oftentimes the hardest to deal with is the AP Lang synthesis essay rubric. Today will focus on it specifically to make sure you know exactly what you’re going to be facing during your test.

What Is a Synthesis Essay AP Lang?

At its core, the AP Lang synthesis essay is a pretty straightforward part of the AP Lang test. It might look pretty similar to the reading section of the exam. However, simply finding the right information isn’t enough. When writing a synthesis essay, you should not only gather the data but also distill it into your personal opinion.

This fine line may seem difficult to spot, but it is there. And it’s that small difference that can make or break your exam run. So try to follow the steps one by one and not lose focus. Writing a good synthesis essay is as easy as following the rules. If you feel this task is too difficult for you, you can leave us your ' write an essay for me ' request and we will do it for you.

AP Lang Synthesis Essay Outline

Looking through AP Lang essay examples, you might notice that the overall structure doesn’t really differ too much from your standard essay outline. You have your introduction, your body, and your conclusion. But the important thing to note is where your arguments are supposed to come from.

You’re not supposed to just go off on a rant. The task requires you to base your supporting evidence on at least three sources. And you will have to ensure your essay has solid roots. Here’s what a basic AP Lang exam synthesis essay outline should look like:

  • Introduction

Provide sufficient context for the topic you are about to cover. You can do a quick overview of prevailing opinions you have grasped while browsing through your source materials.

Write a short and compelling thesis statement. This will be your ground zero for the rest of the essay. So make sure it reflects your opinion. What is a thesis statement you can read in our special article.

  • Body Paragraphs

Dedicate at least one paragraph to every source you’re using. Start with presenting the evidence you have gathered from that source and go on to explain how it formed your opinion on the topic and why it should be considered.

Quickly go through your line of reasoning and reinforce what you have already covered. Finish up with restating your thesis as you’re supposed to logically arrive at it after all the evidence you have presented. That’s how you write a conclusion properly.

Different Forms and Types of Synthesis Essay: Explanatory vs. Argumentative Synthesis Essays

When it comes to writing a synthesis essay AP Lang, there are several types of essays you should consider. The most common ones are the AP Lang argument essay and explanatory essay. The clues as to how each of them should look are hidden within their names but let’s go over them to clear any confusion.

An explanatory essay’s goal is to go over a certain topic, discuss it in detail, and ultimately show a high level of understanding of the said topic. You don’t necessarily have to get into a heated argument with the reader trying to convince them of something. All you need to do is create an impartial overview.

On the other hand, an argumentative essay has to do with personal opinions. And while there is a time and a place for bias, it still has to be as impartial and factual as possible. When proving your point, try not to devolve into emotional arguments but stick to logic and cold truths. This will make your argument way more solid.

Synthesis Essay Structure

In the general case, you don’t really need to look for a synthesis essay AP Lang example to get a solid grasp on how its structure should look like. You can safely fall back on your high school essay writing knowledge, and you’ll be mostly safe.

What you should pay attention to is your writing style and content. A synthesis essay is identified less by its structure and more by the way you form and present your arguments to the reader. It’s when you get a specific essay type (like an argumentative essay) that you should pay attention to slight changes in format.

Argument Essay Structure

The best way to understand argumentative essay structure is to study any well-written AP Lang argument essay example. Standard AP Lang essays have very distinctive features that are very easy to spot and emulate. They follow a very rigid form and employ specific rhetorical devices that you’ll be able to pick up after you analyze them once or twice quickly.

How Many Paragraphs Should an AP Lang Synthesis Essay Be?

The number of paragraphs in an AP Lang synthesis essay can indeed make a difference. Your arguments should be concise and pointed. Spreading them out throughout many paragraphs may seem like a good idea to fill in the space. But it’s actually detrimental to your final score. You can get a basic understanding of what your score is going to be using an AP Lang score calculator.

The same goes for too few paragraphs. Don’t even try to squeeze your entire line of thought into a single body paragraph. Generally, the minimum number of sources you should address is three. Any less, and you are getting a lower score. So try to keep it somewhere in the middle. Three to five body paragraphs is an optimal number. Don’t forget to add an intro and a conclusion to it and you’re all set. A well-written essay has a clear and easily identifiable structure.

How to Write AP Lang Synthesis Essay: Guide

How to Write the AP Lang Synthesis Essay

In order to write a decent essay, all you have to do is follow these simple steps. Performing a rhetorical analysis essay example, AP Lang won’t give you insight into how it was built from the ground up. But looking at this list might.

Step 1. Read the Prompt

It may sound like a no-brainer. But it’s actually more important than you can imagine. Don’t skip right past this step. It’s very easy to misunderstand the task under stress. And if you do slip up in the beginning - the entirety of your work after that is wasted.

Step 2. Analyze the Sources Carefully

The same goes for your sources. Take your time reading them. Try to spot every smallest detail, as even a single one can help you better incorporate your evidence into the body of your essay. You can begin outlining the general points of your essay in your head at this point.

Step 3. Come Up with a Strong Thesis Statement

Your thesis statement is the baseline of your writing. Make it short and clear. Try not to overthink it too much.

Step 4. Fill in Your Essay Outline

Start filling out your outline step by step. You don’t have to go from top to bottom. If you feel like you’re struggling - skip to the next part and return to the problem paragraph later. The use of rhetorical devices AP Lang is also pretty important. So once you flesh out your essay a bit, spend some time trying to come up with the perfect wording.

Step 5. Finalize

The first finished version of your essay is a draft. Don’t be hasty to turn it in. Read over it a couple of times. Make sure everything is in order. You can switch some of the parts around or rewrite some sections if you have the time. Ideally, at this stage you should have enough time to eliminate all grammatical errors that may still be present in your essay. Polish it to perfection.

Useful Tips

Here are some useful tips that might make the writing process a bit easier for you:

  • Use either APA or Chicago style to cite your sources
  • Have a schedule to understand how much time you have for each section
  • Leave as much time as you can for editing and proofreading
  • You can never over study the source material. Spend as much time as you can reading into it
  • Don’t linger on the surface of your essay subject. Dive in and show your complex understanding of the material
  • Avoid using private life anecdotes to support your case unless the essay type specifically allows it. These don’t make for a convincing argument.
  • Use as many supporting arguments as you can but make sure they are actually solid and relevant to your thesis
  • Check with your thesis from time to time. The entirety of your text should align with it

Need help with academic deadlines?

Falling back on your deadlines? Use our term paper writing services to relieve you while you get back on your feet.

AP Lang Essay Prompts

Here are some interesting prompts. Some of them could be found in the previous iterations of the test; you may have spotted them in some of the AP Lang essay examples. Others are there to help you practice for the AP Lang exam 2022.

  • The John F. Kennedy Presidential Library and Museum, dedicated in 1979, was founded in memory of the president and contained archives pertaining to his administration. On June 24, 1985, then President Ronald Reagan joined members of the Kennedy family at a fundraising event to help the Kennedy Library Foundation create an endowment to fund and support the presidential library. The following is an excerpt from the speech Reagan gave at that event. Read the passage carefully. Write an essay that analyzes the rhetorical choices Reagan makes to achieve his purpose of paying tribute to John F. Kennedy.
  • On August 29, 2009, then-President Barack Obama delivered a eulogy at the funeral of Senator Ted Kennedy in Boston, Massachusetts. Kennedy served in the United States Senate from 1962 until his death. Obama served with him in the Senate from 2005 until Obama was elected president in 2008. The following is an excerpt from Obama’s speech. Read the passage carefully. Write an essay that analyzes the rhetorical choices Obama makes to achieve his purpose of praising and memorializing Kennedy.
  • On April 9, 1964, Claudia “Lady Bird” Johnson, who was at the time the First Lady of the United States, gave the following speech at the first-anniversary luncheon of the Eleanor Roosevelt Memorial Foundation. The foundation is a nonprofit division of the Franklin D. Roosevelt Presidential Library dedicated to the works of former First Lady Eleanor Roosevelt, who passed away in 1962. Read the passage carefully. Write an essay that analyzes the rhetorical choices Johnson makes to achieve her purpose of paying tribute to Eleanor Roosevelt.

In your response, you should do the following:

• Respond to the prompt with a thesis that analyzes the writer’s rhetorical choices.

• Select and use evidence to support your line of reasoning.

• Explain how the evidence supports your line of reasoning.

• Demonstrate an understanding of the rhetorical situation.

• Use appropriate grammar and punctuation in communicating your argument.

AP Lang Essay Example

Here is a decent if a bit shortened, AP Lang rhetorical analysis essay example you can use for reference.

Literature to Prepare for AP Lang

How to Write the AP Lang Synthesis Essay

And here is a list of some great AP Lang books that will help you prepare for the exam. Not all of them are immediately useful, but most will help you enhance your writing and analytical abilities to get a better score in the end.

  • The Odyssey
  • Don Quixote
  • A Midsummer Night's Dream
  • Pride and Prejudice
  • Wuthering Heights
  • Oliver Twist
  • Crime and Punishment
  • Adventures of Huckleberry Finn

If you have thoughts of "who could do my paper for me," do not forget that you can contact us. Or, if you have a finished paper and you need to make edits to it, leave us a ' rewrite my essay ' request and we will do it as soon as possible.

Daniel Parker

Daniel Parker

is a seasoned educational writer focusing on scholarship guidance, research papers, and various forms of academic essays including reflective and narrative essays. His expertise also extends to detailed case studies. A scholar with a background in English Literature and Education, Daniel’s work on EssayPro blog aims to support students in achieving academic excellence and securing scholarships. His hobbies include reading classic literature and participating in academic forums.

synthesis essay 2022 ap lang

is an expert in nursing and healthcare, with a strong background in history, law, and literature. Holding advanced degrees in nursing and public health, his analytical approach and comprehensive knowledge help students navigate complex topics. On EssayPro blog, Adam provides insightful articles on everything from historical analysis to the intricacies of healthcare policies. In his downtime, he enjoys historical documentaries and volunteering at local clinics.

Related Articles

How to Find Credible Sources

Calculate for all schools

Your chance of acceptance, your chancing factors, extracurriculars, writing a synthesis essay for ap lang.

I have to write a synthesis essay this week for my AP Lang class, but I'm not 100% sure what exactly that entails. Can someone explain what the process is like and maybe share some tips on how to write a strong synthesis essay?

Sure, a synthesis essay is a type of essay that requires you to use multiple sources to create an argument. In an AP Lang synthesis essay, you'll typically be provided with the sources and will need to analyze them, identify the main ideas, and then connect those ideas to your central argument or thesis.

Here's a step-by-step breakdown of how to write a strong synthesis essay for AP Lang:

1. Understand the prompt : Carefully read the prompt and make sure you understand what's being asked. Identify the main issue or topic you'll be addressing.

2. Develop a thesis statement : Your thesis statement should make a clear and specific claim about the topic. It should be an arguable point that you'll support using evidence from the sources provided.

3. Read and analyze the sources : Read through each source, taking notes on key ideas, evidence, and arguments related to your thesis. Think about how each source contributes to your argument and look for connections between sources.

4. Outline your essay : Create an outline that organizes your ideas and evidence into a logical sequence. Typically, a synthesis essay will have an introduction, body paragraphs, and conclusion. Each body paragraph should focus on a specific point related to your thesis and include evidence from multiple sources to support the point.

5. Write the introduction : Begin with a hook to engage the reader and introduce the topic. Then, give some brief background information about the issue before presenting your thesis statement.

6. Write the body paragraphs : In each body paragraph, start with a topic sentence that introduces the main point of the paragraph. Then, provide evidence and analysis from the sources to support the point, making sure to cite your sources according to the citation style required by your instructor. Try to incorporate evidence from multiple sources and emphasize the connections between them.

7. Address counterarguments : It's important to address any opposing views or counterarguments to show that you've considered different perspectives. You can either refute counterarguments or concede some validity while still maintaining your overall argument.

8. Write the conclusion : Summarize your main points and restate your thesis statement in a new way. You can also suggest further implications of your argument or propose a call to action. Avoid introducing new information in the conclusion.

Some additional tips for writing a strong synthesis essay:

- Make sure your analysis connects effectively to your thesis statement.

- Clearly explain how the evidence from the sources supports your argument.

- Use appropriate transitions to guide your reader through the essay and help them understand the connections between ideas.

- Edit and revise your essay for clarity, grammar, and punctuation.

Good luck with your AP Lang synthesis essay!

About CollegeVine’s Expert FAQ

CollegeVine’s Q&A seeks to offer informed perspectives on commonly asked admissions questions. Every answer is refined and validated by our team of admissions experts to ensure it resonates with trusted knowledge in the field.

Recently viewed courses

Recently viewed.

Find Your Dream School

This site uses various technologies, as described in our Privacy Policy, for personalization, measuring website use/performance, and targeted advertising, which may include storing and sharing information about your site visit with third parties. By continuing to use this website you consent to our Privacy Policy and Terms of Use .

   COVID-19 Update: To help students through this crisis, The Princeton Review will continue our "Enroll with Confidence" refund policies. For full details, please click here.

Enter your email to unlock an extra $25 off an SAT or ACT program!

By submitting my email address. i certify that i am 13 years of age or older, agree to recieve marketing email messages from the princeton review, and agree to terms of use., guide to the ap english language and composition exam.

AP English Language Exam

Can you apply the rhetorical triangle to a piece of writing? Are you able to argue a position? The AP ® English Language and Composition exam tests topics and skills discussed in your Advanced Placement English Language course. If you score high enough, your AP English Language score could earn you college credit!

Check out our AP English Language Guide for what you need to know about the exam:

  • Exam Overview
  • Sections and Question Types
  • How to Prepare

What’s on the AP English Language & Composition Exam?

The College Board is very detailed in what they require your AP teacher to cover in his or her AP English Language & Composition course. The exam tests your abilities to understand how authors use rhetoric and language to convey their purpose. Students are also expected to apply these techniques to their own writing and research projects. Some of the major skills tested include the ability to:

  • Identify an author’s purpose and intended audience
  • Recognize rhetorical devices and strategies in an author’s work
  • Demonstrate understanding of citations in research papers
  • Apply these skills and techniques to their own writing
  • Create and organize an argument defended with evidence and reasoning
  • Plan, write, and revise cogent, well-written essays

Check out our line of AP guides  for a comprehensive content review.

AP English Language Sections & Question Types

The AP English Language & Composition exam is 3 hours and 15 minutes long and consists of two sections: a multiple-choice section and a free response section.

Read More: Review for the exam with our  AP English Language Crash Course 

Multiple-Choice

For AP English Language multiple-choice questions, you are presented with two Reading Passages and three Writing passages. The two Reading passages are nonfiction passages taken from all sorts of works. The idea is to get you to focus on rhetorical devices, figures of speech and intended purposes, under rigid time constraints and with material you haven’t seen before. The three Writing passages are student-produced essays. The idea is to get you to revise the essay that help the writer accomplish his or her goal.

Free Response

The AP English Language section contains three essay prompts: a synthesis essay, a rhetorical analysis essay, and an argument essay.

  • Synthesis essay: You’ll be given a scenario and tasked with writing a response using at least three of six or seven short accompanying sources for support.
  • Rhetorical analysis essay: Asks you to analyze the techniques an author uses, and discuss how they contribute to the author’s purpose.
  • Argument essay: Presents a claim or assertion in the prompt and then asks you to argue a position based on your own knowledge, experience, or reading.

How to Interpret AP English Language Scores

AP scores are reported from 1 to 5. Colleges are generally looking for a 4 or 5 on the AP English Language exam, but some may grant AP credit for a 3. Each test is curved so scores vary from year to year. Here’s how AP English Lang students scored on the May 2022 test:

Source: College Board

How can I prepare?

AP classes are great, but for many students they’re not enough! For a thorough review of AP English Language content and strategy, pick the AP prep option that works best for your goals and learning style.

  • AP Exams  

Explore Colleges For You

Explore Colleges For You

Connect with our featured colleges to find schools that both match your interests and are looking for students like you.

Career Quiz

Career Quiz

Take our short quiz to learn which is the right career for you.

Connect With College Coaches

Get Started on Athletic Scholarships & Recruiting!

Join athletes who were discovered, recruited & often received scholarships after connecting with NCSA's 42,000 strong network of coaches.

Best 389 Colleges

Best 389 Colleges

165,000 students rate everything from their professors to their campus social scene.

SAT Prep Courses

1400+ course, act prep courses, free sat practice test & events,  1-800-2review, free digital sat prep try our self-paced plus program - for free, get a 14 day trial.

synthesis essay 2022 ap lang

Free MCAT Practice Test

I already know my score.

synthesis essay 2022 ap lang

MCAT Self-Paced 14-Day Free Trial

synthesis essay 2022 ap lang

Enrollment Advisor

1-800-2REVIEW (800-273-8439) ext. 1

1-877-LEARN-30

Mon-Fri 9AM-10PM ET

Sat-Sun 9AM-8PM ET

Student Support

1-800-2REVIEW (800-273-8439) ext. 2

Mon-Fri 9AM-9PM ET

Sat-Sun 8:30AM-5PM ET

Partnerships

  • Teach or Tutor for Us

College Readiness

International

Advertising

Affiliate/Other

  • Enrollment Terms & Conditions
  • Accessibility
  • Cigna Medical Transparency in Coverage

Register Book

Local Offices: Mon-Fri 9AM-6PM

  • SAT Subject Tests

Academic Subjects

  • Social Studies

Find the Right College

  • College Rankings
  • College Advice
  • Applying to College
  • Financial Aid

School & District Partnerships

  • Professional Development
  • Advice Articles
  • Private Tutoring
  • Mobile Apps
  • Local Offices
  • International Offices
  • Work for Us
  • Affiliate Program
  • Partner with Us
  • Advertise with Us
  • International Partnerships
  • Our Guarantees
  • Accessibility – Canada

Privacy Policy | CA Privacy Notice | Do Not Sell or Share My Personal Information | Your Opt-Out Rights | Terms of Use | Site Map

©2024 TPR Education IP Holdings, LLC. All Rights Reserved. The Princeton Review is not affiliated with Princeton University

TPR Education, LLC (doing business as “The Princeton Review”) is controlled by Primavera Holdings Limited, a firm owned by Chinese nationals with a principal place of business in Hong Kong, China.

PrepScholar

Choose Your Test

Sat / act prep online guides and tips, expert guide to the ap language and composition exam.

author image

Advanced Placement (AP)

article-71342_640.jpg

With the 2023 AP English Language and Composition exam happening on Tuesday, May 9, it's time to make sure that you're familiar with all aspects of the exam. In this article, I'll give a brief overview of the test, do a deeper dive on each of the sections, discuss how the exam is scored, offer some strategies for studying, and finally wrap up with some essential exam day tips.

Exam Overview

The AP Language and Composition exam tests your rhetorical and composition skills. Essentially, how do authors construct effective arguments in their writing? What tools do they use? How can you use those tools to craft effective writing yourself? That is the essence of rhetorical analysis.

The exam has two parts: the first section is an hour-long, 45 question multiple-choice section. It includes five sets of questions, each based on a passage or passages. In this section, there will be 23-25 rhetorical analysis questions which test your rhetorical skills. There will also be 20-22 writing questions which require you to consider revisions to the texts you're shown.

The second section is free response. It starts with a 15-minute reading period, and then you'll have 120 minutes to write three analytical essays:

  • One essay where you synthesize several provided texts to create an argument
  • One essay where you analyze a nonfiction passage for its rhetorical construction
  • One essay where you create an original argument in response to a prompt.

You will have about 40 minutes to write each essay, but no one will prompt you to move from essay to essay—you can structure the 120 minutes as you wish.

In the next sections I'll go over each section of the exam more closely—first multiple choice, and then free response.

The AP English Language and Composition Multiple-Choice

The multiple-choice section tests you on two main areas. The first is how well you can read and understand nonfiction passages for their use of rhetorical devices and tools. The second is how well you can "think like a writer" and make revisions to texts in composition questions.

You will be presented with five passages, about which you will receive a small amount of orienting information, e.g. "This passage is excerpted from a collection of essays on boating" or "This passage is excerpted from an essay written in 19th-century Haiti." Each passage will be followed by a set of questions.

There are, in general, eight question types you can expect to encounter on the multiple-choice section of the exam. I've taken my examples from the sample questions in the " Course and Exam Description ."

eight-1316133_640.jpg

Magic eight-ball says there are eight types of multiple-choice questions!

Type 1: Reading Comprehension

These questions are focused on verifying that you understood what a certain part of the passage was saying on a concrete, literal level. You can identify these questions from phrases like "according to" "refers," etc. The best way to succeed on these questions is to go back and re-read the part of the passage referred to very carefully.

Comprehension.png

Type 2: Implication

These questions take reading comprehension one step further—they are primarily focused on what the author is implying without directly coming out and saying it. These questions will have a correct answer, though, based on evidence from the passage. Which interpretation offered in the answers does the passage most support? You can identify questions like these from words like "best supported," ‘"implies," "suggests," "inferred," and so on.

implies.png

Type 3: Overall Passage and Author Questions

These questions ask about overall elements of the passage or the author, such as the author's attitude on the issue discussed, the purpose of the passage, the passage's overarching style, the audience for the passage, and so on.

You can identify these questions because they won't refer back to a specific moment in the text. For these questions, you'll need to think of the passage from a "bird's-eye view" and consider what all of the small details together are combining to say.

3overall_passage.png

Type 4: Relationships Between Parts of the Text

Some questions will ask you to describe the relationship between two parts of the text, whether they are paragraphs or specific lines. You can identify these because they will usually explicitly ask about the relationship between two identified parts of the text, although sometimes they will instead ask about a relationship implicitly, by saying something like "compared to the rest of the passage."

4relationship.png

Type 5: Interpretation of Imagery/Figurative Language

These questions will ask you about the deeper meaning or implication of figurative language or imagery that is used in the text. Essentially, why did the author choose to use this simile or this metaphor? What is s/he trying to accomplish?

You can generally identify questions like this because the question will specifically reference a moment of figurative language in the text. However, it might not be immediately apparent that the phrase being referenced is figurative, so you may need to go back and look at it in the passage to be sure of what kind of question you are facing.

5imagery.png

Type 6: Purpose of Part of the Text

Still other questions will ask you to identify what purpose a particular part of the text serves in the author's larger argument. What is the author trying to accomplish with the particular moment in the text identified in the question?

You can identify these questions because they will generally explicitly ask what purpose a certain part of the text serves. You may also see words or phrases like "serves to" or "function."

6purpose_of_part.png

Type 7: Rhetorical Strategy

These questions will ask you to identify a rhetorical strategy used by the author. They will often specifically use the phrase "rhetorical strategy," although sometimes you will be able to identify them instead through the answer choices, which offer different rhetorical strategies as possibilities.

7rhetorical_strategy.png

Type 8: Composition

This is the newest question type, first seen in the 2019/2020 school year. For these questions, the student will need to act as though they are the writer and think through different choices writers need to make when writing or revising text.

These questions can involve changing the order of sentences or paragraphs, adding or omitting information to strengthen an argument or improve clarity, making changes to draw reader attention, and other composition-based choices.

body_composition

Some very important stylish effects going on here.

The AP English Language and Composition Free Response

The free response section has a 15-minute reading period. After that time, you will have 120 minutes to write three essays that address three distinct tasks.

Because the first essay involves reading sources, it is suggested that you use the entire 15-minute reading period to read the sources and plan the first essay. However, you may want to glance at the other questions during the reading period so that ideas can percolate in the back of your mind as you work on the first essay.

Essay One: Synthesis

For this essay, you will be briefly oriented on an issue and then given anywhere from six to seven sources that provide various perspectives and information on the issue. You will then need to write an argumentative essay with support from the documents.

If this sounds a lot like a DBQ , as on the history AP exams, that's because it is! However, this essay is much more argumentative in nature—your goal is to persuade, not merely interpret the documents.

Example (documents not included, see 2022 free response questions ):

body-AP-Literature-synthesis

Essay Two: Rhetorical Analysis

In the second essay, you'll be presented with an excerpt from a nonfiction piece that advances an argument and asked to write an essay analyzing the rhetorical strategies used to construct the passage's argument. You will also be given some orienting information—where the passage was excerpted from, who wrote it, its approximate date, where it was published (if at all), and to whom it was directed.

Example (excerpt not included, see 2022 free response questions ):

body-AP-literature-Question-2

Essay Three: Argument

In the third essay, you will be presented with an issue and asked to write a persuasive essay taking a position on the issue. You will need to support your position with evidence from your "reading, experience, and observations."

body-AP-lit-Question-3

This doesn't look like a very well-constructed argument.

How The AP Language and Composition Exam Is Scored

The multiple-choice section of the exam is worth 45% of your score, and the free-response section is worth the other 55%. So each of the three free-response essays is worth about 18% of your score.

As on other APs, your raw score will be converted to a scaled score of 1-5. This exam has a relatively low 5 rate. Only 10% of test takers received a 5 in 2022 , although 56% of students received a score of 3 or higher.

In terms of how the raw score is obtained, the multiple-choice section is similar to other AP multiple-choice sections: you receive a point for every question you answer correctly, and there is no penalty for guessing.

The grading rubrics for the free-response questions were revamped in 2019. They are scored using analytic rubrics instead of holistic rubrics. For each free-response question, you will be given a score from 0-6. The rubrics assess three major areas:

#1: Thesis (0 to 1 points): Is there a thesis, and does it properly respond to the prompt?

#2: Evidence and Commentary (0 to 4 points): Does the essay include supporting evidence and analysis that is relevant, specific, well organized, and supports the thesis?

#3: Sophistication (0 to 1 points): Is the essay well-crafted and does it show a sufficiently nuanced understanding of the prompt?

Each scoring rubric broadly assesses these three factors. However, each task is also different in nature, so the rubrics do have some differences. I'll go over each rubric—and what it really means—for you here.

Synthesis Essay Rubrics

EVIDENCE AND COMMENTARY

SOPHISTICATION

dough-196235_640.jpg

Time to synthesize this dough into some cookies.

Rhetorical Analysis Essay Rubrics

magnifying-glass-914922_640.png

Examine your texts closely!

Argumentative Essay Rubrics

playing-puppies-790638_640.jpg

The best kind of frenzy is a puppy frenzy!

AP English Language Prep Tips

Unlike its cousin, the AP English Literature and Composition exam, the AP Language and Composition exam (and course) have very little to do with fiction or poetry. So some students used to more traditional English classes may be somewhat at a loss as to what to do to prepare.

Luckily for you, I have a whole slate of preparation tips for you!

Read Nonfiction—In a Smart Way

A major thing you can do to prepare for the AP Lang and Comp exam is to read nonfiction— particularly nonfiction that argues a position , whether explicitly (like an op-ed) or implicitly (like many memoirs and personal essays). Read a variety of non-fiction genres and topics, and pay attention to the following:

  • What is the author's argument?
  • What evidence do they use to support their position?
  • What rhetorical techniques and strategies do they use to build their argument?
  • Are they persuasive? What counterarguments can you identify? Do they address them?

Thinking about these questions with all the reading you do will help you hone your rhetorical analysis skills.

Learn Rhetorical Terms and Strategies

Of course, if you're going to be analyzing the nonfiction works you read for their rhetorical techniques and strategies, you need to know what those are! You should learn a robust stable of rhetorical terms from your teacher, but here's my guide to the most important AP Language and Composition terms .

  • We've compiled a list of 20 rhetorical devices you should know.
  • A heroic individual from Riverside schools in Ohio uploaded this aggressively comprehensive list of rhetorical terms with examples. It's 27 pages long, and you definitely shouldn't expect to know all of these for the exam, but it's a useful resource for learning some new terms.
  • Another great resource for learning about rhetorical analysis and how rhetorical devices are actually used is the YouTube Channel Teach Argument , which has videos rhetorically analyzing everything from Taylor Swift music videos to Super Bowl commercials. It's a fun way to think about rhetorical devices and get familiar with argumentative structures.
  • Finally, a great book—which you might already use in your class—is " They Say, I Say. " This book provides an overview of rhetoric specifically for academic purposes, which will serve you well for AP preparation and beyond.

You also need to practice argumentative and persuasive writing. In particular, you should practice the writing styles that will be tested on the exam: synthesizing your own argument based on multiple outside sources, rhetorically analyzing another piece of writing in-depth, and creating a completely original argument based on your own evidence and experience.

You should be doing lots of writing assignments in your AP class to prepare, but thoughtful, additional writing will help. You don't necessarily need to turn all of the practice writing you do into polished pieces, either—just writing for yourself, while trying to address some of these tasks, will give you a low-pressure way to try out different rhetorical structures and argumentative moves, as well as practicing things like organization and developing your own writing style.

once-upon-a-time-719174_640.jpg

Not the most auspicious start to an argumentative essay.

Practice for the Exam

Finally, you'll need to practice specifically for the exam format. There are sample multiple-choice questions in the " AP Course and Exam Description ," and old free-response questions on the College Board website.

Unfortunately, the College Board hasn't officially released any complete exams from previous years for the AP English Language and Composition exam, but you might be able to find some that teachers have uploaded to school websites and so on by Googling "AP Language complete released exams." I also have a guide to AP Language and Composition practice tests .

Once you're prepped and ready to go, how can you do your best on the test?

Looking for help studying for your AP exam? Our one-on-one online AP tutoring services can help you prepare for your AP exams. Get matched with a top tutor who got a high score on the exam you're studying for!

AP Language and Composition Test Day Tips

Here are four key tips for test-day success.

board-1193334_640.jpg

You are one hundred percent success!

Interact With the Text

When you are reading passages, both on the multiple-choice section and for the first two free-response questions, interact with the text! Mark it up for things that seem important, devices you notice, the author's argument, and anything else that seems important to the rhetorical construction of the text. This will help you engage with the text and make it easier to answer questions or write an essay about the passage.

Think About Every Text's Overarching Purpose and Argument

Similarly, with every passage you read, consider the author's overarching purpose and argument. If you can confidently figure out what the author's primary assertion is, it will be easier to trace how all of the other aspects of the text play into the author's main point.

Plan Your Essays

The single most important thing you can do for yourself on the free-response section of the AP English Language exam is to spend a few minutes planning and outlining your essays before you start to write them.

Unlike on some other exams, where the content is the most important aspect of the essay, on the AP Language Exam, organization, a well-developed argument, and strong evidence are all critical to strong essay scores. An outline will help you with all of these things. You'll be able to make sure each part of your argument is logical, has sufficient evidence, and that your paragraphs are arranged in a way that is clear and flows well.

Anticipate and Address Counterarguments

Another thing you can do to give your free responses an extra boost is to identify counterarguments to your position and address them within your essay. This not only helps shore up your own position, but it's also a fairly sophisticated move in a timed essay that will win you kudos with AP graders.

envelope-392962_640.jpg

Address counterarguments properly or they might get returned to sender!

Key Takeaways

The AP Language and Composition exam tests your rhetorical skills. The exam has two sections.

The first section is an hour-long, 45 question multiple-choice test based on the rhetorical techniques and composition choices.

The second section is a two-hour free-response section (with a 15-minute initial reading period) with three essay questions: one where you must synthesize given sources to make an original argument, one where you must rhetorically analyze a given passage, and one where you must create a wholly original argument about an issue with no outside sources given.

You'll receive one point for every correct answer on the multiple-choice section of the exam, which is worth 45% of your score. The free-response section is worth 55% of your score. For each free-response question, you'll get a score based on a rubric from 0-6. Your total raw score will be converted to a scaled score from 1-5.

Here are some test prep strategies for AP Lang:

#1 : Read nonfiction with an eye for rhetoric #2 : Learn rhetorical strategies and techniques #3 : Practice writing to deploy rhetorical skills #4 : Practice for the exam!

Here are some test-day success tips:

#1 : Interact with each passage you encounter! #2 : Consider every text's overarching purpose and argument. #3 : Keep track of time #4 : Plan your essays #5 : Identify and address counterarguments in your essays.

With all of this knowledge, you're ready to slay the AP English Language and Composition beast!

animal-1299749_640.png

Noble knight, prepare to slay the AP dragon!

What's Next?

Want more AP Lang review? We have a complete collection of released AP Language practice tests , as well as a list of the AP Lang terms you need to know and a guide to the multiple choice section .

Taking the AP Literature exam? Check out our ultimate guide to the AP English Literature test and our list of AP Literature practice tests .

Taking other AP exams? See our Ultimate Guides to AP World History , AP US History , AP Chemistry , AP Biology , AP World History , and AP Human Geography .

Need more AP prep guidance? Check out how to study for AP exams and how to find AP practice tests .

Want to build the best possible college application?   We can help.   PrepScholar Admissions combines world-class admissions counselors with our data-driven, proprietary admissions strategies. We've guided thousands of students to get into their top choice schools, from state colleges to the Ivy League. We know what kinds of students colleges want to admit and are driven to get you admitted to your dream schools. Learn more about PrepScholar Admissions to maximize your chance of getting in:

These recommendations are based solely on our knowledge and experience. If you purchase an item through one of our links, PrepScholar may receive a commission.

author image

Ellen has extensive education mentorship experience and is deeply committed to helping students succeed in all areas of life. She received a BA from Harvard in Folklore and Mythology and is currently pursuing graduate studies at Columbia University.

Ask a Question Below

Have any questions about this article or other topics? Ask below and we'll reply!

Improve With Our Famous Guides

  • For All Students

The 5 Strategies You Must Be Using to Improve 160+ SAT Points

How to Get a Perfect 1600, by a Perfect Scorer

Series: How to Get 800 on Each SAT Section:

Score 800 on SAT Math

Score 800 on SAT Reading

Score 800 on SAT Writing

Series: How to Get to 600 on Each SAT Section:

Score 600 on SAT Math

Score 600 on SAT Reading

Score 600 on SAT Writing

Free Complete Official SAT Practice Tests

What SAT Target Score Should You Be Aiming For?

15 Strategies to Improve Your SAT Essay

The 5 Strategies You Must Be Using to Improve 4+ ACT Points

How to Get a Perfect 36 ACT, by a Perfect Scorer

Series: How to Get 36 on Each ACT Section:

36 on ACT English

36 on ACT Math

36 on ACT Reading

36 on ACT Science

Series: How to Get to 24 on Each ACT Section:

24 on ACT English

24 on ACT Math

24 on ACT Reading

24 on ACT Science

What ACT target score should you be aiming for?

ACT Vocabulary You Must Know

ACT Writing: 15 Tips to Raise Your Essay Score

How to Get Into Harvard and the Ivy League

How to Get a Perfect 4.0 GPA

How to Write an Amazing College Essay

What Exactly Are Colleges Looking For?

Is the ACT easier than the SAT? A Comprehensive Guide

Should you retake your SAT or ACT?

When should you take the SAT or ACT?

Stay Informed

Follow us on Facebook (icon)

Get the latest articles and test prep tips!

Looking for Graduate School Test Prep?

Check out our top-rated graduate blogs here:

GRE Online Prep Blog

GMAT Online Prep Blog

TOEFL Online Prep Blog

Holly R. "I am absolutely overjoyed and cannot thank you enough for helping me!”

pep

Find what you need to study

2024 AP English Language and Composition Exam Guide

12 min read • august 18, 2023

A Q

Your guide to the 2024 AP English Language and Composition exam

We know that studying for your AP exams can be stressful, but Fiveable has your back! We created a study plan to help you crush your AP English Language and Composition exam. This guide will continue to update with information about the 2024 exams, as well as helpful resources to help you do your best on test day.  Unlock Cram Mode  for access to our cram events—students who have successfully passed their AP exams will answer your questions and guide your last-minute studying LIVE! And don't miss out on unlimited access to our database of thousands of practice questions.

Format of the 2024 AP English Language and Composition exam

This year, all AP exams will cover all units and essay types. The 2024 AP English Language and Composition exam format will be:

Section I: Multiple Choice - 45% of your score

45 questions in 1 hour

Section II: Free Response Section - 55% of your score

2 hours and 15 minutes for:

1 synthesis essay

1 rhetorical analysis essay

1 argument essay

Scoring Rubric for the 2024 AP Lang Essays

Synthesis Essay

1 point for a defensible thesis that responds to the prompt

Evidence and Commentary

Max of 4 points for providing evidence from at least 3 sources that support the line of reasoning AND commentary that explains and analyzes the evidence

Sophistication

1 point any of the following:

Creating a nuanced argument

Showing the limitations of the argument

Making effective rhetorical choices

Employing a style that is vivid and persuasive

Rhetorical Analysis Essay

1 point for a defensible thesis that analyzes rhetorical choices

Max of 4 points for providing specific evidence AND consistently explaining how the evidence relates to the line of reasoning AND showing how the rhetorical choices contribute to the author's message .

1 point for any of the following:

Explaining the significance of the rhetorical choices ( rhetorical situation )

Explaining the complexities of the passage and their purpose

Argument Essay

1 point for a defensible thesis

Max of 4 points for providing specific evidence AND consistently explaining the relevance of that evidence .

Crafting a nuanced argument by identifying complexities

Explaining the limitations of the argument by placing it in a broader context

Making rhetorical choices to improve the argument

Check out our study plan below to find resources and tools to prepare for your AP English Language and Composition exam.

When is the 2024 AP English Language and Composition Exam and How Do I Take It?

How should i prepare for the ap lang exam.

First, take stock of your progress in the course so far. What areas have you excelled and which sections need more focus? Download the AP English Language Cheatsheet PDF - a single sheet that covers everything you need to know at a high level. Take note of your strengths and weaknesses!

Build your study plan to review every unit and question type, but focus most on the areas that need the most improvement and practice. We’ve put together this plan to help you study between now and May. This will cover all of the units and essay types to prepare you for your exam

Practice essays are your best friends! The more essays you write, the more automatic the process will come, and the easier the AP exam will be!

Try some of the past exam questions here

We've put together the study plan found below to help you study between now and May. This will cover all of the units and essay types to prepare you for your exam. Pay special attention to the units that you need the most improvement in.

Study, practice, and review for test day with other students during our live cram sessions via  Cram Mode . Cram live streams will teach, review, and practice important topics from AP courses, college admission tests, and college admission topics. These streams are hosted by experienced students who know what you need to succeed.

Pre-Work: Set Up Your Study Environment

Before you begin studying, take some time to get organized.

🖥 Create a study space.

Make sure you have a designated place at home to study. Somewhere you can keep all of your materials, where you can focus on learning, and where you are comfortable. Spend some time prepping the space with everything you need and you can even let others in the family know that this is your study space. 

📚 Organize your study materials.

Get your notebook, textbook, prep books, or whatever other physical materials you have. Also, create a space for you to keep track of review. Start a new section in your notebook to take notes or start a Google Doc to keep track of your notes. Get yourself set up!

📅 Plan designated times for studying.

The hardest part about studying from home is sticking to a routine. Decide on one hour every day that you can dedicate to studying. This can be any time of the day, whatever works best for you. Set a timer on your phone for that time and really try to stick to it. The routine will help you stay on track.

🏆 Decide on an accountability plan.

How will you hold yourself accountable to this study plan? You may or may not have a teacher or rules set up to help you stay on track, so you need to set some for yourself. First, set your goal. This could be studying for x number of hours or getting through a unit. Then, create a reward for yourself. If you reach your goal, then x. This will help stay focused!

2024 AP Lang Study Guide

🚧 unit 1 foundations of rhetoric: analysis of the rhetorical situation and claims ., big takeaways:.

Unit 1 is an introductory unit that lays the foundations for the reading skills associated with how to understand and analyze complex texts. Skills here include identifying the ASPECTS of a text, analyzing the claim given and the evidence used to support that claim, and determining the function of the “chunks” in the argument. Because the content in this unit is very foundational, it is looped throughout the rest of the course instruction.

Definitely do this:

📚 Read these study guides:

Unit 1 Overview: Claims , Reasoning , and Evidence

1.1 Identifying the purpose and intended audience of a text

1.2 Examining how evidence supports a claim

1.3 Developing paragraphs as part of an effective argument

🎥 Watch these videos:

College Board’s Instructional Video: Overview of The Rhetorical Situation .

Fiveable’s How to Read Like an AP Student .

Rhetorical Analysis Thesis Statements  

Rhetorical Analysis Body Paragraphs

✍️ Practice:

Use the Fiveable ASPECTS Guidesheet to help you break down a complex text.

🗺 Can you identify these rhetorical devices?

You won’t be asked to name drop on the exam, but it can be helpful to use devices when discussing strategies. Try this Quizlet to help prepare.

Unit 2 Foundations of Argument: Analysis of an author’s choices in appeals and evidence

Unit 2 is an introductory unit that builds onto the foundations of rhetorical ASPECTS and moves toward planning and writing your own arguments. This unit focuses on the relationships between subject, speaker, and message, including examination of the structure and purpose of the given argument. The unit then moves into the developing thesis statements and building your own arguments with a clear line of reasoning .

Unit 2 Overview: Organizing Information for a Specific Audience

2.1 Analyzing audience and its relationship to the purpose of an argument

2.2 Building an argument with relevant and strategic evidence

2.3 Developing thesis statements

2.4 Developing structure and integrating evidence to reflect a line of reasoning

College Board’s Instructional Video: Identify Rhetorical Situation in a Pre 20th Century Text .

Fiveable’s video on How to Find Rhetorical Devices  

📰 Check out these articles:

Here’s a list of recommended rhetorical devices with definitions and examples!

Use the Fiveable Rhetorical Precis Guidesheet to help you break down a complex text.

🗺 Can you identify these elements of practical argument?

You won’t be asked to name drop of the exam, but it can be helpful to use devices when discussing strategies. Try this Quizlet to help prepare.

👥 Unit 3 Confluence: Synthesis of multiple sources in argumentation

Unit 3 approaches multiple perspectives in argument through the lens of synthesis (that’s FRQ 1). In this study, you learn to identify effective and faulty reasoning while integrating a variety of evidence from credible resources that is properly cited in an original text.

Unit 3 Overview: Perspectives and How Arguments Relate

3.1 Interpreting character description and perspective

3.2 Identifying and avoiding flawed lines of reasoning

3.3 Introducing and integrating sources and evidence

3.4 Using sufficient evidence for an argument

3.5 Attributing and citing references

3.6 Developing parts of a text with cause-effect and narrative methods

Fiveable’s Introduction into Synthesis Essays and How to Begin Your Argument

College Board’s Instructional Video: Complexity in Argument .

🗺 Can you identify these elements of synthesis?

👀 Unit 4 Reasoning : Analysis of argument from introduction to conclusion

Unit 4 includes a greater depth of focus on the writing of effective arguments -- the line of reasoning created in the introduction, built with modes of discourse, and strengthened in the conclusion. An important note about these skills of argumentation is that they build toward all parts of every FRQ. 

Unit 4 Overview: How writers develop arguments, intros, and conclusion

4.1 Developing and connecting thesis statements and lines of reasoning

4.2 Developing introductions and conclusions

4.3 Adjusting an argument to address new evidence

College Board’s Instructional Video: Understanding a Line of Reasoning .

Fiveable’s Effective Annotations .

Try Fiveable’s Guide to LOR Body Paragraphs .

🗺 Can you identify the rhetorical modes?

You won’t be asked to name drop them on the exam, but it can be helpful to use devices when discussing strategies. Try this Quizlet to help prepare.

🧐 Unit 5 Commentary and Analysis: Analysis of complex argument and intentional rhetoric

In Unit 5, the skills look at the minutiae involved in argumentation: development of the line of reasoning that produces strong commentary and maintains the primary claim through all parts of the writing. To achieve these goals, this unit includes a focus on transitions , modifiers , and qualifications for argumentative perspective .  

Unit 5 Overview

5.1 Maintaining ideas throughout an argument

5.2 Developing commentary throughout paragraphs

5.3 Using modifiers to qualify an argument and convey perspective

5.4 Using transitions

Fiveable’s video on How to Improve Analysis Part 1 and Part 2

As well as how to Embed Quotes into Body Paragraphs  

Rhetorical Analysis Body Paragraphs  

Synthesis Essay Body Paragraphs  

Argument Essay Body Paragraphs

Tara Seale’s adaptation for Creating a Line of Reasoning .

🏃‍♂️ Unit 6 Rhetorical Risks: Analysis of multiple perspectives , bias , and shifts with new evidence

In Unit 6, you will notice a direct link building on the ideas of Unit 3 as this instruction looks at position and perspectives while synthesizing information strategically to support a claim.  For greater depth, this unit moves to modify a current argument to include new evidence .

Unit 6 Overview: Position, Perspective , and Bias

6.1 Incorporating multiple perspectives strategically into an argument

6.2 Recognizing and accounting for bias

6.3 Adjusting an argument to new evidence

6.4 Analyzing tone and shifts in tone

College Board’s Instructional Video: Creating a Nuanced Argument .

Fiveable’s video on Tracking an Author’s Argument  

🚀 Unit 7 Complex Argumentation: Analysis of effective arguments, including concession and refutation

The skills of Unit 7 are about putting all units of study together to look at the complexity of a given argument and the effectiveness of the pieces built into that argument.  Though many teachers will have addressed counterarguments, concessions, and refutations before reaching this unit, those skills are highly scrutinized in this segment of learning.

Unit 7 Overview: Successful and Unsuccessful Arguments

7.1 Examining complexities in issues

7.2 Considering how words, phrases, and clauses can modify and limit an argument

7.3 Examining how counterargument or alternative perspectives affect an argument

7.4 Exploring how sentence development affects an argument

Fiveable’s video on Arguments and Counterarguments  

College Board’s Instructional Video: How Argument Demonstrates Understanding .

Check your progress with Fiveable’s AP Language Skills Matrix .

📝 Unit 8 Style: Analysis of how style influences the audience movement

Unit 8 covers how to understand the influence style has on the audience , and the purpose behind each decision. By analyzing these various tactics, students are able to understand the author’s audience , and how to effectively persuade them. Style is an important part in connecting the rest of the course and understanding how the rhetorical choices and devices are used to accomplish a purpose .

Unit 8 Overview: Stylistic Choices

8.1 Choosing comparisons based on an audience

8.2 Considering how sentence development and word choice affect how the writer is perceived by an audience

8.3 Considering how all choices made in an argument affect the audience

8.4 Considering how style affects an argument

Fiveable’s Analysis of the Mindset of the Audience

College Board’s Instructional video: Analyzing and Understanding the Audience

College Board’s explanation of Elements and Context for Style  

Review this quizlet on Elements of Style for more practice.

✏️ Unit 9 Craft: Creation of your own complex argument with synthesis and rhetoric

The final unit of AP Language and Composition covers how to effectively form your own arguments by acknowledging and understanding complexities to create a nuanced and sophisticated argument. It focuses on your ability to comprehend and connect multiple sources to create a well reasoned, and detailed argument as well as how to add in your own rhetorical devices and choices to make your writing more persuasive and effective.

Unit 9 Overview: Developing a Complex Argument

9.1 Strategically conceding, rebutting, or refuting information

9.2 Crafting an argument through stylistic choices like word choice and description

Fiveable’s video on Creating your own Synthesis Arguments

College Board’s video on Complexities within Arguments and How to Create a Nuanced Argument

Key Terms to Review ( 38 )

Argument Structure

Author's Message

Cause-Effect Method

Comparisons

Conclusions

Counterargument

Introductions

Line of Reasoning

Multiple Perspectives

Narrative Method

Objective Reasoning

Perspective

Qualifications

Rhetorical Choices

Rhetorical Situation

Sentence Development

Stylistic Choices

Subjective Reasoning

Textual Evidence

Thesis Development

Thesis Statement

Tone Shifts

Transitions

Word Choice

Fiveable

Stay Connected

© 2024 Fiveable Inc. All rights reserved.

AP® and SAT® are trademarks registered by the College Board, which is not affiliated with, and does not endorse this website.

What are your chances of acceptance?

Calculate for all schools, your chance of acceptance.

Duke University

Your chancing factors

Extracurriculars.

synthesis essay 2022 ap lang

How to Write the AP Lang Argument Essay + Examples

What’s covered:, what is the ap language argument essay, tips for writing the ap language argument essay, ap english language argument essay examples, how will ap scores impact my college chances.

In 2023, over 550,148 students across the U.S. took the AP English Language and Composition Exam, and 65.2% scored higher than a 3. The AP English Language Exam tests your ability to analyze a piece of writing, synthesize information, write a rhetorical essay, and create a cohesive argument. In this post, we’ll be discussing the best way to approach the argumentative essay section of the test, and we’ll give you tips and tricks so you can write a great essay.

The AP English Language Exam as of 2023 is structured as follows:

Section 1: 45 multiple choice questions to be completed in an hour. This portion counts for 45% of your score. This section requires students to analyze a piece of literature. The questions ask about its content and/or what could be edited within the passage.

Section 2: Three free response questions to be completed in the remaining two hours and 15 minutes. This section counts for 55% of your score. These essay questions include the synthesis essay, the rhetorical essay, and the argumentative essay.

  • Synthesis essay: Read 6-7 sources and create an argument using at least three of the sources.
  • Rhetorical analysis essay: Describe how a piece of writing evokes meaning and symbolism.
  • Argumentative essay: Pick a side of a debate and create an argument based on evidence. In this essay, you should develop a logical argument in support of or against the given statement and provide ample evidence that supports your conclusion. Typically, a five paragraph format is great for this type of writing. This essay is scored holistically from 1 to 9 points.

Do you want more information on the structure of the full exam? Take a look at our in-depth overview of the AP Language and Composition Exam .

Although the AP Language Argument may seem daunting at first, once you understand how the essay should be structured, it will be a lot easier to create cohesive arguments.

Below are some tips to help you as you write the essay.

1. Organize your essay before writing

Instead of jumping right into your essay, plan out what you will say beforehand. It’s easiest to make a list of your arguments and write out what facts or evidence you will use to support each argument. In your outline, you can determine the best order for your arguments, especially if they build on each other or are chronological. Having a well-organized essay is crucial for success.

2. Pick one side of the argument, but acknowledge the other side

When you write the essay, it’s best if you pick one side of the debate and stick with it for the entire essay. All your evidence should be in support of that one side. However, in your introductory paragraph, as you introduce the debate, be sure to mention any merit the arguments of the other side has. This can make the essay a bit more nuanced and show that you did consider both sides before determining which one was better. Often, acknowledging another viewpoint then refuting it can make your essay stronger.

3. Provide evidence to support your claims

The AP readers will be looking for examples and evidence to support your argument. This doesn’t mean that you need to memorize a bunch of random facts before the exam. This just means that you should be able to provide concrete examples in support of your argument.

For example, if the essay topic is about whether the role of the media in society has been detrimental or not, and you argue that it has been, you may talk about the phenomenon of “fake news” during the 2016 presidential election.

AP readers are not looking for perfect examples, but they are looking to see if you can provide enough evidence to back your claim and make it easily understood.

4. Create a strong thesis statement

The thesis statement will set up your entire essay, so it’s important that it is focused and specific, and that it allows for the reader to understand your body paragraphs. Make sure your thesis statement is the very last sentence of your introductory paragraph. In this sentence, list out the key points you will be making in the essay in the same order that you will be writing them. Each new point you mention in your thesis should start a paragraph in your essay.

Below is a prompt and sample student essay from the May 2019 exam . We’ll look at what the student did well in their writing and where they could improve.

Prompt: “The term “overrated” is often used to diminish concepts, places, roles, etc. that the speaker believes do not deserve the prestige they commonly enjoy; for example, many writers have argued that success is overrated, a character in a novel by Anthony Burgess famously describes Rome as a “vastly overrated city,” and Queen Rania of Jordan herself has asserted that “[b]eing queen is overrated.”

Select a concept, place, role, etc. to which you believe that the term “overrated” should be applied. Then, write a well-developed essay in which you explain your judgment. Use appropriate evidence from your reading, experience, or observations to support your argument.

Sample Student Essay #1:

[1] Competition is “overrated.” The notion of motivation between peers has evolved into a source of unnecessary stress and even lack of morals. Whether it be in an academic environment or in the industry, this new idea of competition is harmful to those competing and those around them.

[2] Back in elementary school, competition was rather friendly. It could have been who could do the most pushups or who could get the most imaginary points in a classroom for a prize. If you couldn’t do the most pushups or win that smelly sticker, you would go home and improve yourself – there would be no strong feelings towards anyone, you would just focus on making yourself a better version of yourself. Then as high school rolled around, suddenly applying for college doesn’t seem so far away –GPA seems to be that one stat that defines you – extracurriculars seem to shape you – test scores seem to categorize you. Sleepless nights, studying for the next day’s exam, seem to become more and more frequent. Floating duck syndrome seems to surround you (FDS is where a competitive student pretends to not work hard but is furiously studying beneath the surface just like how a duck furiously kicks to stay afloat). All of your competitors appear to hope you fail – but in the end what do you and your competitor’s gain? Getting one extra point on the test? Does that self-satisfaction compensate for the tremendous amounts of acquired stress? This new type of “competition” is overrated – it serves nothing except a never-ending source of anxiety and seeks to weaken friendships and solidarity as a whole in the school setting.

[3] A similar idea of “competition” can be applied to business. On the most fundamental level, competition serves to be a beneficial regulator of prices and business models for both the business themselves and consumers. However, as businesses grew increasingly greedy and desperate, companies resorted to immoral tactics that only hurt their reputations and consumers as a whole. Whether it be McDonald’s coupons that force you to buy more food or tech companies like Apple intentionally slowing down your iPhone after 3 years to force you to upgrade to the newest device, consumers suffer and in turn speak down upon these companies. Similar to the evolved form of competition in school, this overrated form causes pain for all parties and has since diverged from the encouraging nature that the principle of competition was “founded” on.

The AP score for this essay was a 4/6, meaning that it captured the main purpose of the essay but there were still substantial parts missing. In this essay, the writer did a good job organizing the sections and making sure that their writing was in order according to the thesis statement. The essay first discusses how competition is harmful in elementary school and then discusses this topic in the context of business. This follows the chronological order of somebody’s life and flows nicely.

The arguments in this essay are problematic, as they do not provide enough examples of how exactly competition is overrated. The essay discusses the context in which competition is overrated but does not go far enough in explaining how this connects to the prompt.

In the first example, school stress is used to explain how competition manifests. This is a good starting point, but it does not talk about why competition is overrated; it simply mentions that competition can be unhealthy. The last sentence of that paragraph is the main point of the argument and should be expanded to discuss how the anxiety of school is overrated later on in life. 

In the second example, the writer discusses how competition can lead to harmful business practices, but again, this doesn’t reflect the reason this would be overrated. Is competition really overrated because Apple and McDonald’s force you to buy new products? This example could’ve been taken one step farther. Instead of explaining why business structures—such as monopolies—harm competition, the author should discuss how those particular structures are overrated.

Additionally, the examples the writer used lack detail. A stronger essay would’ve provided more in-depth examples. This essay seemed to mention examples only in passing without using them to defend the argument.

It should also be noted that the structure of the essay is incomplete. The introduction only has a thesis statement and no additional context. Also, there is no conclusion paragraph that sums up the essay. These missing components result in a 4/6.

Now let’s go through the prompt for a sample essay from the May 2022 exam . The prompt is as follows:

Colin Powell, a four-star general and former United States Secretary of State, wrote in his 1995 autobiography: “[W]e do not have the luxury of collecting information indefinitely. At some point, before we can have every possible fact in hand, we have to decide. The key is not to make quick decisions, but to make timely decisions.”

Write an essay that argues your position on the extent to which Powell’s claim about making decisions is valid. 

In your response you should do the following:

  • Respond to the prompt with a thesis that presents a defensible position. 
  • Provide evidence to support your line of reasoning. 
  • Explain how the evidence supports your line of reasoning. 
  • Use appropriate grammar and punctuation in communicating your argument.

Sample Student Essay #2:

Colin Powell, who was a four star general and a former United States Secretary of State. He wrote an autobiography and had made a claim about making decisions. In my personal opinion, Powell’s claim is true to full extent and shows an extremely valuable piece of advice that we do not consider when we make decisions.

Powell stated, “before we can have every possible fact in hand we have to decide…. but to make it a timely decision” (1995). With this statement Powell is telling the audience of his autobiography that it does not necessarily matter how many facts you have, and how many things you know. Being able to have access to everything possible takes a great amount of time and we don’t always have all of the time in the world. A decision has to be made with what you know, waiting for something else to come in while trying to make a decision whether that other fact is good or bad you already have a good amount of things that you know. Everyone’s time is valuable, including yours. At the end of the day the decision will have to be made and that is why it should be made in a “timely” manner.

This response was graded for a score of 2/6. Let’s break down the score to smaller points that signify where the student fell short.

The thesis in this essay is clearly outlined at the end of the first paragraph. The student states their agreement with Powell’s claim and frames the rest of their essay around this stance. The success in scoring here lies in the clear communication of the thesis and the direction the argument will take. It’s important to make the thesis statement concise, specific, and arguable, which the student has successfully done.

While the student did attempt to provide evidence to support their thesis, it’s clear that their explanation lacks specific detail and substance. They referenced Powell’s statement, but did not delve into how this statement has proven true in specific instances, and did not provide examples that could bring the argument to life.

Commentary is an essential part of this section’s score. It means explaining the significance of the evidence and connecting it back to the thesis. Unfortunately, the student’s commentary here is too vague and does not effectively elaborate on how the evidence supports their argument.

To improve, the student could use more concrete examples to demonstrate their point and discuss how each piece of evidence supports their thesis. For instance, they could discuss specific moments in Powell’s career where making a timely decision was more valuable than waiting for all possible facts. This would help illustrate the argument in a more engaging, understandable way.

A high score in the “sophistication” category of the grading rubric is given for demonstrating a complex understanding of the rhetorical situation (purpose, audience, context, etc.), making effective rhetorical choices, or establishing a line of reasoning. Here, the student’s response lacks complexity and sophistication. They’ve simply agreed with Powell’s claim and made a few general statements without providing a deeper analysis or effectively considering the rhetorical situation.

To increase sophistication, the student could explore possible counterarguments or complexities within Powell’s claim. They could discuss potential drawbacks of making decisions without all possible facts, or examine situations where timely decisions might not yield the best results. By acknowledging and refuting these potential counterarguments, they could add more depth to their analysis and showcase their understanding of the complexities involved in decision-making.

The student could also analyze why Powell, given his background and experiences, might have come to such a conclusion, thus providing more context and showing an understanding of the rhetorical situation.

Remember, sophistication in argumentation isn’t about using fancy words or complicated sentences. It’s about showing that you understand the complexity of the issue at hand and that you’re able to make thoughtful, nuanced arguments. Sophistication shows that you can think critically about the topic and make connections that aren’t immediately obvious.

Now that you’ve looked at an example essay and some tips for the argumentative essay, you know how to better prepare for the AP English Language and Composition Exam.

While your AP scores don’t usually impact your admissions chances , colleges do care a lot about your course rigor. So, taking as many APs as you can will certainly boost your chances! AP scores can be a way for high-performing students to set themselves apart, particularly when applying to prestigious universities. Through the process of self-reporting scores , you can show your hard work and intelligence to admissions counselors.

That said, the main benefit of scoring high on AP exams comes once you land at your dream school, as high scores can allow you to “test out” of entry-level requirements, often called GE requirements or distribution requirements. This will save you time and money.

To understand how your course rigor stacks up, check out CollegeVine’s free chancing engine . This resource takes your course rigor, test scores, extracurriculars, and more, to determine your chances of getting into over 1600 colleges across the country!

Related CollegeVine Blog Posts

synthesis essay 2022 ap lang

IMAGES

  1. AmStud

    synthesis essay 2022 ap lang

  2. 💣 Ap language synthesis prompts. AP English Language and Composition

    synthesis essay 2022 ap lang

  3. 💄 Synthesis essay template. How to Write the AP Lang Synthesis Essay

    synthesis essay 2022 ap lang

  4. How to Write an AP Lang Synthesis Essay: A Complete Guide

    synthesis essay 2022 ap lang

  5. How to write a good synthesis essay ap lang

    synthesis essay 2022 ap lang

  6. 💐 How to write a synthesis essay ap english. Synthesis Essay Example

    synthesis essay 2022 ap lang

VIDEO

  1. League Champions and their cooler versions

  2. Chris Synthesis Essay Revision

  3. Synthesis essay template

  4. how to write a synthesis and argument essay in 4 minutes

  5. Writing a Synthesis Essay, Composite Summary and Referencing During Examination/ UGRC210 PastQuo2024

  6. Synthesis Activities

COMMENTS

  1. AP English Language and Composition Exam Questions

    Download free-response questions from this year's exam and past exams along with scoring guidelines, sample responses from exam takers, and scoring distributions. If you are using assistive technology and need help accessing these PDFs in another format, contact Services for Students with Disabilities at 212-713-8333 or by email at ssd@info ...

  2. How to Write the AP Lang Synthesis Essay with Example

    AP Lang Exam Basics. The AP Lang exam is separated into two sections. In the first section, students have one hour to answer a series of 45 multiple-choice questions. Here, about half of the questions are based on passages students read. The other half are focused on the best revision techniques.

  3. How to Write the AP Lang Synthesis Essay + Example

    The AP Lang synthesis essay is the first of three essays included in the Free Response section of the AP Lang exam. The exam presents 6-7 sources that are organized around a specific topic, with two of those sources purely visual, including a single quantitative source (like a graph or pie chart).

  4. How to Write a Perfect Synthesis Essay for the AP Language Exam

    Paragraph 1: The prompt presents and briefly explains the topic that you'll be writing your synthesis essay about. That topic is the concept of eminent domain. Paragraph 2: The prompt presents a specific claim about the concept of eminent domain in this paragraph: Eminent domain is productive and beneficial.This paragraph instructs you to decide whether you want to defend, challenge, or ...

  5. AP English Language Exam Practice: Synthesis Study Plan

    Resources you need to improve your Synthesis essay on the AP English Language and Composition exam. Includes revelant readings and practice problems. Note: For best results, click to highlight and copy/paste this list into your Fiveable Rooms Task Card to automatically create individual tasks. Jumpstart your studying in 5 seconds!

  6. Acing the AP® English Language and Composition Synthesis Essay

    Last Updated On: March 1, 2022. The newest section of the AP® English Language and Composition Exam, the synthesis essay, is one of three essays you will be completing during the examination's 2-hour free-response period. However, you'll also have a 15-minute reading and planning period just for this essay, and if you use this time to plan ...

  7. How to Write the AP Lang Synthesis Essay + Essay Template

    So let's have a look with our coursework writing services team at what your AP Lang exam 2022 might look like. ... At its core, the AP Lang synthesis essay is a pretty straightforward part of the AP Lang test. It might look pretty similar to the reading section of the exam. However, simply finding the right information isn't enough.

  8. 2022 Live Review 4

    In this AP Daily: Live Review session, we will review key skills, knowledge, and strategies necessary for success when writing the synthesis essay. We will r...

  9. AP Lang

    Section II of the AP English Language and Composition exam includes three free-response questions that you must answer in 2 hours and 15 minutes. This guide will focus on Question 1 of Section II of the exam, the Synthesis question. As with all AP exams with free-response questions, the Synthesis question has its own rubric and scoring that we ...

  10. Writing a synthesis essay for AP Lang

    Sure, a synthesis essay is a type of essay that requires you to use multiple sources to create an argument. In an AP Lang synthesis essay, you'll typically be provided with the sources and will need to analyze them, identify the main ideas, and then connect those ideas to your central argument or thesis. Here's a step-by-step breakdown of how to write a strong synthesis essay for AP Lang: 1.

  11. Guide to the AP English Language and Composition Exam

    AP scores are reported from 1 to 5. Colleges are generally looking for a 4 or 5 on the AP English Language exam, but some may grant AP credit for a 3. Each test is curved so scores vary from year to year. Here's how AP English Lang students scored on the May 2022 test: Score. Meaning.

  12. Expert Guide to the AP Language and Composition Exam

    How The AP Language and Composition Exam Is Scored. The multiple-choice section of the exam is worth 45% of your score, and the free-response section is worth the other 55%. So each of the three free-response essays is worth about 18% of your score. As on other APs, your raw score will be converted to a scaled score of 1-5.

  13. PDF AP Scoring Rubric for Question 1: Synthesis Essay

    AP ® English Language and Composition. Scoring Rubric for Question 1: Synthesis Essay. 0 POINTS. 1 POINT: For any of the following: No defensible thesis Simple restatement of prompt only Summary of topic with no clear claim States an apparent fact rather than a defensible claim Off-topic

  14. AP Lang Exam Guide

    The 2024 AP English Language and Composition exam format will be: Section I: Multiple Choice - 45% of your score. 45 questions in 1 hour. Section II: Free Response Section - 55% of your score. 2 hours and 15 minutes for: 1 synthesis essay. 1 rhetorical analysis essay. 1 argument essay.

  15. How to Write the AP Lang Argument Essay + Examples

    2. Pick one side of the argument, but acknowledge the other side. When you write the essay, it's best if you pick one side of the debate and stick with it for the entire essay. All your evidence should be in support of that one side. However, in your introductory paragraph, as you introduce the debate, be sure to mention any merit the ...

  16. PDF AP English Language and Composition

    Synthesis Essay 6 points Since the early 2000s, the United States government and a number of corporations have sponsored initiatives to improve education in the STEM disciplines: science, technology, engineering, and mathematics.

  17. Synthesis

    Synthesis (Argument with Sources) Prompts. Happiness Prompt. Cyberbully Prompt. Social Media/Facebook Prompt. School Cafeteria Prompt. Sleep Deprivation Prompt. Zombie Apocalypse Prompt. Technology in the 21st Century Prompt.